You are on page 1of 9

Test No.

IAS Prelims - 2020 25


PTS2020/PPP-25/012020/26

CSAT - 2

RE
GENERAL MENTAL ABILITY

O
Answer Key
SC
Q. 1 (c) Q. 11 (a) Q. 21 (d) Q. 31 (b) Q. 41 (b)

Q. 2 (a) Q. 12 (d) Q. 22 (a) Q. 32 (b) Q. 42 (b)


GS

Q. 3 (d) Q. 13 (c) Q. 23 (c) Q. 33 (b) Q. 43 (b)

Q. 4 (c) Q. 14 (a) Q. 24 (d) Q. 34 (b) Q. 44 (c)

Q. 5 (d) Q. 15 (b) Q. 25 (b) Q. 35 (c) Q. 45 (d)

Q. 6 (d) Q. 16 (b) Q. 26 (a) Q. 36 (c) Q. 46 (c)

Q. 7 (b) Q. 17 (b) Q. 27 (a) Q. 37 (a) Q. 47 (c)

Q. 8 (b) Q. 18 (a) Q. 28 (b) Q. 38 (a) Q. 48 (d)

Q. 9 (c) Q. 19 (b) Q. 29 (d) Q. 39 (d) Q. 49 (c)

Q. 10 (b) Q. 20 (c) Q. 30 (c) Q. 40 (b) Q. 50 (b)

1 PTS2020/PPP-25/012020/26
PTS2020/PPP-36/122019/04
1. Correct Option (c) Number of people who vote for Q = 60

Explanation : On the last day, Number of people who


broke their promise, to vote for P
Inflation rate for the year 2017 & 2018
= (15% of 40) = 6 voters
= 8% each
Number of people who broke their
The rate of increase in price of sugar will
promise, to vote for Q = (25% of 60)
be = 10%.
= 15 voters
As, price of sugar on Jan 1, 2017 is
So the number of votes P have
Rs. 20
= (40 – 6 + 15) = 49 votes
Then, it will be Rs. 22 on Jan 1, 2018
and Rs. 24.20 on Jan 1, 2019. The number of votes Q have
= (60 – 15 + 6) = 51 votes.
2. Correct Option: (a)
Hence, margin of victory for
Explanation : Q = (51 – 49) = 2
Initially I – E = S (I = Income, 2% of total = 4;
E = expenditure, S = saving) Hence the total number of voters is 200.
10000 – 6000 = 4000 (saving) The total number of voters for P is

Now, I = 11000 and E = 7200. 49 × 2 = 98

So saving = I – E = 3800.
5. Correct Option (d)
(4000 – 3800) Explanation :
= 5%
4000
Let the ratio of content = p : q
Quantity of A in the mixture is
3. Correct Option: (d)

Explanation : 5 1
= p+ q
6 4
By reverse side calculation,
32 – 16 = 16
Quantity of B in the mixture =
16 × 2 = 32
32 + 8 = 40 Therefore, the required mixture = 2 : 1
Hence, input is 40.

4. Correct Option: (c)

Explanation :
Let the total number of votes = 100

2 By solving,
of 100 = 40
5
p 5
Initially, there were 40 of these, who We get =
q 2
promised to vote for P.

2
6. Correct Option (d) and share of B = Q
Explanation :
Let the turn off time of A = p min. Given
Now we have case when the pipe was
closed: we know that for the last 5 min B P 8
worked alone Hence =
Q 3
5 1
hence = rd of the tank is filled by 9. Correct Option (c)
15 3
B alone. Explanation :
1 Let Mrs. Joshi’s saving = 3s
If rd of the tank is filled by B alone,
3 Let Mrs. Joshi’s expenditure = 2s

2 Therefore her initial total income


then rd of the tank is filled by A & B becomes 5s
3
together A and B can full the tank in Now her income increases by 10 %
So New income will be 5.5s
9 1
= = . Her expenditure increases by 20 % and
60 T
therefore Her new expenditure
20 = 3s × 120% = 3.6s
That means, in mins A and B can
3 Therefore her new saving is
fill the complete tank = 5.5s – 3.6s = 1.9s
Hence for the time for which A is on
Percentage change in savings
2
= rd of the tank filled
3
= × 100 = 5%
2s
2 20 40
= × = . 10. Correct Option (b)
3 3 9
Hence answer is none of these Explanation :
Total ways from A to C via B = 3 × 2 = 6
7. Correct Option (b)
Total ways from A to C directly = 2
Explanation :
Hence, Total ways = 6 + 2 = 8
Let the share of Ruma is p Then the
share of Ashish is (61200 – p) 11. Correct Option (a)
Therefore
Explanation :
Let the age of new students is a, and b
px = (61200– p) years.
Then,
240 –10 + a + b = (20 –1 + 2) × 12
px = (61200 –p) 230 + a + b = 252
a + b = 22.........(1)
By solving, we get p = Rs. 30000
Given,
8. Correct Option (b) a – b = 4..........(2)
Solve (1) and (2),
Explanation :
b = 9 years
Let the share of A = P

3
12. Correct Option (d) 17. Correct Option (b)
Explanation : Explanation :
Here we are going to solve this sum by
1 1
ACB = AOB = × 90º = 45º making equations.
2 2
So let the number of cards Shyam has
be M and let Ram have I cards with him.
13. Correct Option (c)
Now get back to the conditions, according
to question , if Shyam gives Ram x cards,
= then the equation :
l + x = 4(M – x) i.e. 4M – I = 5x.
clearly, the least fraction among options On the other hand if Ram gives Shyam
35 x cards, then we have the equation :
given is to make a whole number.
48
I – x = 3(M + x) i.e. I – 3M = 4x.
Solving the two equations we get,
14. Correct Option (a)
M = 9x and I = 31x.
Explanation :
The only option is 31. In a pack of 52
Last digit of R1 = 1=1
cards the only possible value for I could
Last digit of R2 = 2=2+1=3
be 31.
Last digit of R3 = 3=3+2+1=6
| | 18. Correct Option (a)
| | Explanation :
| |
Let the person buy c oranges, 2c pears.
Last digit of R10 = 10 = 55 Therefore, bananas is >3c.
In any case, total cost will be
15. Correct Option (b) cost of oranges + cost of pears + cost of
Explanation : bananas.
Number of persons, who read something (c ×1) + [2c × 0.5] + (2 × something
greater than 3c).
= 175
Thus, Total cost 2c + (2× something
Now, min number of persons who read
either and of the papers greater than 3c).

= 175 – (40 + 50) =85 Here, the total cost is an even number.
Thus Answer should be an even number.
Hence, maximum number of people who
read both the papers = 40 Taking c =1, the total minimum cost we
get is 10.

16. Correct Option (b) Thus the only option that satisfies the
constraints is 34.
Explanation :
Let the number of buffaloes be x and 19. Correct Option (b)
number of ducks be y.
Then, number of legs = 4x + 2y, Explanation :

number of heads = x + y He is getting Rs. 1200 for sure. The rest


Rs. 600 is received in proportion of 12:4
given, 4x + 2y = 2(x + y) + 24
2x = 24 (Commission : bonus.

x =12 Hence he receives Rs 150 as bonus.

4
25. Correct Option (b) Now, share of B: share of A, C, D=1 : 3

Explanation : So, if we divide Rs. 600 in the ratio 1 : 3,


we get 150 and 450.
If the number of employees is 100,
so share of B = Rs. 150
then men = 60, women = 40.
and share of C: share of A, B, D=1 : 4
Engineers = 48.
So, if we divide 600 in the ratio 1:4, we
2 get 120 and 480
66.67% = so share of C= Rs. 120
3
Hence, share of D will be
2
Engineers who are men = 48 32 . = 600 – (200 + 150 +120)
3
= 600 – 470 = Rs. 130.
So, women engineers = 48 – 32 = 16.
Hence net percentage of women who are Answers (29-31) :
16 Explanation :
engineers = 100 40% .
40 Amount on Saturday morning
= (200 + 25) × 2 = Rs. 450;
26. Correct Option (a) Amount on Friday morning
Explanation : = (450 + 200) × 2 = Rs. 1300;

Relative speed = 8 – 6 = 2 km/h Amount on Thursday morning

Length of track = speed × time = (1300 + 200) × 2 = Rs. 3000;


Amount on Wednesday morning
30 = (3000 + 200) × 2 = Rs. 6400;
=2× = 1 km.
60
Amount on Tuesday morning
= (6400 + 200) × 2 = Rs.13200;
27. Correct Option (a)
Amount on Monday morning
Explanation :
= (13200 + 200) × 2 = Rs. 26,800.
Square numbers – a, b, c
(a + 5) + (b + 5)+ (c + 5) = 44 29. Correct Option (d)
a + b + c = 44 – 15 = 29 Explanation :
Possible values of a, b, c = 4, 9, 16 [Out Amount spent on Wednesday
of 1, 4, 9, 16, 25]
The age of Roni and Anika after two 6400
=
years = 29; 2
The age of Roni and Meenu after four
= Rs. 3200.
years = 28
Alternatively, amount spent on
Hence, Quantity I > Quantity II.
Wednesday
= amount available on Thursday morning
28. Correct Option (b)
+ 200
Explanation : = 3000 + 200 = Rs.3200.
According to the question
share of A : share of B, C, D=1 : 2 30. Correct Option (c)
So, if we divide Rs. 600 in the ratio 1 : 2, Explanation :
we get 200 and 400. Amount on Monday morning
so share of A = Rs. 200 = (13200 + 200) × 2 = Rs. 26,800.

6
31. Correct Option (b)
= 36 3 3 cm3.
Explanation :
Total expenditure = 26,800 – 25
35. Correct Option (c)
= Rs 26,775.
Explanation :

32. Correct Option (b) Sanjeev beats Motiani by 10 seconds


Motiani covers a distance of 1000 m in
Explanation :
200 seconds.
Time lost in 48 hrs. 48 1.5 = 72 minutes
He covers a distance of 5 m in 1 second
As the watch owner adjusts 36 minutes, The distance Motiani could have
effective time lost in 48 hours = 36
covered in 10 seconds = 50 m
minutes
Sanjeev can beat Motiani by 50 m.
For the watch to show the right time, it
should loose 12 hrs.
36. Correct Option (c)

= 40 days Explanation :
36
Let M, W and C be the weekly wages for
a man, a woman and a child respectively.
33. Correct Option (b)
20M + 30W + 36C = 780 — (I)
Explanation : Since their wages are proportional to the
A regular year has exactly 52 full weeks work done by them per week,
and 1 day. M:W:C=3:2:1
For maximum saturdays and Sundays, i.e. M = 3C and W = 2C
year has to be started with Saturday. Using this in equation (I)
So maximum required days = 52 × 2 + 1 156C = 780 or C = 5, W = 10, M = 15
= 105 In 4 weeks (one month)
15 men will earn: 4 × 15 × 15 = 900
34. Correct Option (b) 21 women will earn: 4 × 21 × 10 = 840
Explanation : 30 children will earn: 4 × 30 × 5 = 600
The height of the box is the diameter of Total wages = 900 + 840 + 600 = Rs. 2340
the sphere
radius of sphere = 3 cm 37. Correct Option (a)

4 Explanation :
Volume of sphere=
3 Since x2y2 > 0, yz < 0.

= 36 cm3
38. Correct Option (a)
2 Explanation :
Side of hexagon = cms
3
Volume of box

1
= cm3
2
Required ways = ADB, ACEIB, ACEFB,
Required volume = AHEFB, AHEIB, AGB

7
39. Correct Option (d)
Given = 66
Explanation : 2

n(n – 1) = 132
Speed of the first train = m/sec n = 12

= 30 m/sec. 43. Correct Option (b)


Explanation :
Speed of the second train = m/sec
This occurs for values such as 103-013;
213 – 123; 324 – 234 etc.
= 20 m/sec.
Where it can be seen that the value of X
Speed = (30 + 20) m/sec = 50m/sec.
is more than Y.
The possible pairs of values for X and Y
Required time = sec = 12 sec are-(1, 0) ( 2, 1) (3, 2)…….(9, 8)
50
a total of nine pairs of values.

40. Correct Option (b)


44. Correct Option (c)
Explanation :
Explanation :
Probability of choosing a leap year is less
New no. of rollers
1
than because the years ending with
Required difference = 81 – 56 = 25 4
‘00’ may not be a leap year. So,
41. Correct Option (b) probability of not a leap year is greater

Explanation : then .
x2 – y2 = (x + y) (x – y)
Since 17 is a prime no. 45. Correct Option (d)

Hence it can be uniquely factorised as Explanation :


17 × 1. Let he mix 3 kg, 4 kg and 5 kg of dry
So. (x + y) (x – y) = 17 × 1 fruits

x + y = 17 & x – y = 1 Therefore the cost will be

By solving above equations, we get = (3 × 80 + 4 × 100 + 5 × 120) = 1240

x=9&y=8 Now he sells at 50% profit

x2 + y2 = 81 + 64 = 145 Therefore selling price of 12 kg


=1240 × 1.5 = 1860
42. Correct Option (b) So the selling price of 1 kg

Explanation : 1860
= = Rs. 155
Let no. of students = n 12

8
46. Correct Option (c)
PR2
C.I.2 – S.I.2 = = 10 .... (i)
Explanation : 1002
From the given data we come to know
that one value would be negative and PR3 PR2
C.I.3 – S.I.3 = + 3 = 60 .... (ii)
one value would be positive. 1003 1002

Using the conditions 3 < x < 4 and From (i) & (ii), we have

– 9 < y < – 8 , the value of y would be PR3 PR2 PR2


negative and the value of x would be + 3 =
1003 1002 1002
positive.

Let us analyze the options:


60 = 10
We can see that since y has been squared
in option a, its value becomes positive.
R
The value of xy2 would be positive and 6= +3
100
will not be the minimum.
R
From (b) and (c), x2y and 5xy would give 3=
negative values but we cannot find the
100
specific negative and positive values  R = 300%
On comparing (a) and (c), we find that
49. Correct Option (c)
x2 < 5x in 2 < x < 3.
Explanation :
Since the value of y is negative therefore
1
x2y > 5xy (A – 20) = (B + 20) ---------(i)
9
Therefore 5xy would give the minimum
value. and A+20 = B – 20 -----------(ii)
from the above two equations,
47. Correct Option (c) A comes out to be 30.
Explanation :
50. Correct Option (b)
Explanation :
Suppose Jinny lived for x years,
Therefore by given condition

So total age of Jinny comes out to be 84


years.
So, third melon is the best. So Jinny’s age when she got married

48. Correct Option (d) =

Explanation :
= = 33 years.
We know that ,

You might also like